\chapter*{Lösungen der Übungsaufgaben} \addcontentsline{toc}{chapter}{Lösungen der Übungsaufgaben} \begin{solution}[\ref{ub1:aufg1}] \textbf{Teilaufgabe a)} Es gilt: \begin{enumerate}[label=(\roman*)] \item $\emptyset, X \in \fT_X$. \item $\fT_X$ ist offensichtlich unter Durchschnitten abgeschlossen, d.~h. es gilt für alle $U_1, U_2 \in \fT_X: U_1 \cap U_2 \in \fT_X$. \item Auch unter beliebigen Vereinigungen ist $\fT_X$ abgeschlossen, d.~h. es gilt für eine beliebige Indexmenge $I$ und alle $U_i \in \fT_X$ für alle $i \in I: \bigcup_{i \in I} U_i \in \fT_X$ \end{enumerate} Also ist $(X, \fT_X)$ ein topologischer Raum. \textbf{Teilaufgabe b)} Wähle $x=1, y=0$. Dann gilt $x \neq y$ und die einzige Umgebung von $x$ ist $X$. Da $y=0 \in X$ können also $x$ und $y$ nicht durch offene Mengen getrennt werden. $(X, \fT_X)$ ist also nicht hausdorffsch. \textbf{Teilaufgabe c)} Nach Bemerkung \ref{Trennungseigenschaft} sind metrische Räume hausdorffsch. Da $(X, \fT_X)$ nach (b) nicht hausdorffsch ist, liefert die Kontraposition der Trennungseigenschaft, dass $(X, \fT_X)$ kein metrischer Raum sein kann. \end{solution} \begin{solution}[\ref{ub1:aufg4}] \textbf{Teilaufgabe a)} \textbf{Beh.:} $\forall a \in \mdz: \Set{a}$ ist abgeschlossen. Sei $a \in \mdz$ beliebig. Dann gilt: \todo[inline]{Hat jemand diesen Beweis?} \textbf{Teilaufgabe b)} \textbf{Beh.:} $\Set{-1, 1}$ ist nicht offen \textbf{Bew.:} durch Widerspruch Annahme: $\Set{-1, 1}$ ist offen. Dann gibt es $T \subseteq \fB$, sodass $\bigcup_{M \in T} M = \Set{-1, 1}$. Aber alle $U \in \fB$ haben unendlich viele Elemente. Auch endlich viele Schnitte von Elementen in $\fB$ haben unendlich viele Elemente $\Rightarrow$ keine endliche nicht-leere Menge kann in dieser Topologie offen sein $\Rightarrow \Set{-1,1}$ ist nicht offen. $\qed$ \textbf{Beh.:} Es gibt unendlich viele Primzahlen. \textbf{Bew.:} durch Widerspruch Annahme: Es gibt nur endlich viele Primzahlen $p \in \mdp$ Dann ist \[\mdz \setminus \Set{-1, +1} \overset{\text{FS d. Arithmetik}}= \bigcup_{p \in \mdp} U_{0,p}\] endlich. Das ist ein Widerspruch zu $|\mdz|$ ist unendlich und $|\Set{-1,1}|$ ist endlich. $\qed$ \end{solution} \begin{solution}[\ref{ub2:aufg4}] \begin{enumerate}[label=(\alph*)] \item \underline{Beh.:} Die offenen Mengen von $P$ sind Vereinigungen von Mengen der Form \[\prod_{j \in J} U_j \times \prod_{i \in \mdn, i \neq j} P_i\] wobei $J \subseteq \mdn$ endlich und $U_j \subseteq P_j$ offen ist. \begin{beweis} Nach Definition der Produkttopologie bilden Mengen der Form \[\prod_{i \in J} U_j \times \prod_{\stackrel{i \in \mdn}{i \notin J}} P_i, \text{ wobei } J \subseteq \mdn \text{ endlich und } U_j \subseteq P_j \text{offen } \forall{j \in J}\] eine Basis der Topologie. Damit sind die offenen Mengen von $P$ Vereinigungen von Mengen der obigen Form. $\qed$ \end{beweis} \item \underline{Beh.:} Die Zusammenhangskomponenten von $P$ sind alle einpunktig.\xindex{Total Unzusammenhängend} \begin{beweis} Es seinen $x,y \in P$ und $x$ sowie $y$ liegen in der gleichen Zusammenhangskomponente $Z \subseteq P$. Da $Z$ zusammenhängend ist und $\forall{i \in I}: p_i : P \rightarrow P_i$ ist stetig, ist $p_i(Z) \subseteq P_i$ zusammenhängend für alle $i \in \mdn$. Die zusammenhängenden Mengen von $P_i$ sind genau $\Set{0}$ und $\Set{1}$, d.~h. für alle $i \in \mdn$ gilt entweder $p_i(Z) \subseteq \Set{0}$ oder $p_i(Z) \subseteq \Set{1}$. Es sei $z_i \in \Set{0,1}$ so, dass $p_i(Z) \subseteq \Set{z_i}$ für alle $i \in \mdn$. Dann gilt also: \[\underbrace{p_i(x)}_{= x_i} = z_i = \underbrace{p_i(y)}_{= y_i} \forall i \in \mdn\] Somit folgt: $x = y \qed$ \end{beweis} \end{enumerate} \end{solution}